Difference between revisions of "2013 AIME I Problems/Problem 7"
(→Solutions) |
|||
Line 3: | Line 3: | ||
==Solution 1== | ==Solution 1== | ||
+ | Let the height of the box be <math>x</math>. | ||
+ | |||
+ | After using the Pythagorean Theorem three times, we can quickly see that the sides of the triangle are 10, <math>\sqrt{\left(\frac{x}{2}\right)^2 + 64}</math>, and <math>\sqrt{\left(\frac{x}{2}\right)^2 + 36}</math>. Since the area of the triangle is <math>30</math>, the altitude of the triangle from the base with length <math>10</math> is <math>6</math>. | ||
+ | |||
+ | Considering the two triangles created by the altitude, we use the Pythagorean theorem twice to find the lengths of the two line segments that make up the base of <math>10</math>. | ||
+ | |||
+ | We find: | ||
+ | <cmath>10 = \sqrt{\left(28+x^2/4\right)}+x/2</cmath> | ||
+ | |||
+ | Solving for <math>x</math> gives us <math>x=\frac{36}{5}</math>. Since <math>gcd(36,5)=1</math>, therefore: | ||
+ | <cmath>m+n=\boxed{041}</cmath> | ||
+ | |||
+ | ==Solution 2== | ||
We may use vectors. Let the height of the box be <math>2h</math>. Without loss of generality, let the front bottom left corner of the box be <math>(0,0,0)</math>. Let the center point of the bottom face be <math>P_1</math>, the center of the left face be <math>P_2</math> and the center of the front face be <math>P_3</math>. | We may use vectors. Let the height of the box be <math>2h</math>. Without loss of generality, let the front bottom left corner of the box be <math>(0,0,0)</math>. Let the center point of the bottom face be <math>P_1</math>, the center of the left face be <math>P_2</math> and the center of the front face be <math>P_3</math>. | ||
Line 20: | Line 33: | ||
<cmath>m+n=\boxed{041}</cmath> | <cmath>m+n=\boxed{041}</cmath> | ||
− | ==Solution | + | ==Solution 3== |
Let the height of the box be <math>x</math>. | Let the height of the box be <math>x</math>. | ||
After using the Pythagorean Theorem three times, we can quickly see that the sides of the triangle are 10, <math>\sqrt{(x/2)^2 + 64}</math>, and <math>\sqrt{(x/2)^2 + 36}</math>. Therefore, we can use Heron's formula to set up an equation for the area of the triangle. | After using the Pythagorean Theorem three times, we can quickly see that the sides of the triangle are 10, <math>\sqrt{(x/2)^2 + 64}</math>, and <math>\sqrt{(x/2)^2 + 36}</math>. Therefore, we can use Heron's formula to set up an equation for the area of the triangle. | ||
− | The | + | The semiperimeter is (10 + <math>\sqrt{(x/2)^2 + 64}</math> + <math>\sqrt{(x/2)^2 + 36}</math>)/2 |
900 = <math>\frac{1}{2}</math>((10 + <math>\sqrt{(x/2)^2 + 64}</math> + <math>\sqrt{(x/2)^2 + 36}</math>)/2)((10 + <math>\sqrt{(x/2)^2 + 64}</math> + <math>\sqrt{(x/2)^2 + 36}</math>)/2 - 10)((10 + <math>\sqrt{(x/2)^2 + 64}</math> + <math>\sqrt{(x/2)^2 + 36}</math>)/2 - <math>\sqrt{(x/2)^2 + 64}</math>)((10 + <math>\sqrt{(x/2)^2 + 64}</math> + <math>\sqrt{(x/2)^2 + 36}</math>)/2 - <math>\sqrt{(x/2)^2 + 36}</math>). | 900 = <math>\frac{1}{2}</math>((10 + <math>\sqrt{(x/2)^2 + 64}</math> + <math>\sqrt{(x/2)^2 + 36}</math>)/2)((10 + <math>\sqrt{(x/2)^2 + 64}</math> + <math>\sqrt{(x/2)^2 + 36}</math>)/2 - 10)((10 + <math>\sqrt{(x/2)^2 + 64}</math> + <math>\sqrt{(x/2)^2 + 36}</math>)/2 - <math>\sqrt{(x/2)^2 + 64}</math>)((10 + <math>\sqrt{(x/2)^2 + 64}</math> + <math>\sqrt{(x/2)^2 + 36}</math>)/2 - <math>\sqrt{(x/2)^2 + 36}</math>). |
Revision as of 21:30, 31 March 2013
Problem 7
A rectangular box has width inches, length inches, and height inches, where and are relatively prime positive integers. Three faces of the box meet at a corner of the box. The center points of those three faces are the vertices of a triangle with an area of square inches. Find .
Solution 1
Let the height of the box be .
After using the Pythagorean Theorem three times, we can quickly see that the sides of the triangle are 10, , and . Since the area of the triangle is , the altitude of the triangle from the base with length is .
Considering the two triangles created by the altitude, we use the Pythagorean theorem twice to find the lengths of the two line segments that make up the base of .
We find:
Solving for gives us . Since , therefore:
Solution 2
We may use vectors. Let the height of the box be . Without loss of generality, let the front bottom left corner of the box be . Let the center point of the bottom face be , the center of the left face be and the center of the front face be .
We are given that the area of the triangle is . Thus, by a well known formula, we note that We quickly attain that and (We can arbitrarily assign the long and short ends due to symmetry)
Computing the cross product, we find:
Thus:
Solution 3
Let the height of the box be .
After using the Pythagorean Theorem three times, we can quickly see that the sides of the triangle are 10, , and . Therefore, we can use Heron's formula to set up an equation for the area of the triangle.
The semiperimeter is (10 + + )/2
900 = ((10 + + )/2)((10 + + )/2 - 10)((10 + + )/2 - )((10 + + )/2 - ).
Solving, we get .
See also
2013 AIME I (Problems • Answer Key • Resources) | ||
Preceded by Problem 6 |
Followed by Problem 8 | |
1 • 2 • 3 • 4 • 5 • 6 • 7 • 8 • 9 • 10 • 11 • 12 • 13 • 14 • 15 | ||
All AIME Problems and Solutions |